Module 6 - HESI Safety & Infection Control

Pataasin ang iyong marka sa homework at exams ngayon gamit ang Quizwiz!

A nurse enters the laundry room to empty a bag of dirty linen and discovers a fire in a laundry basket. What action should the nurse take first? Extinguishing the fire Confining the fire Activating the fire alarm Running for the fire extinguisher

Activating the fire alarm Rationale: The first priority in the event of a fire is rescuing the clients in immediate danger. In this situation, no clients are in immediate danger. The next step is to activate the fire alarm. The nurse then confines the fire by closing all doors and, finally, extinguishes the fire.

A hospitalized client, experiencing confusion, is at risk of falling because she continually tries to climb out of bed. Which of these safety devices that the nurse might suggest is the least restrictive? Ambularm Elbow Belt Wrist

Ambularm Rationale: The Ambularm device, worn on the leg, signals when the client's leg is in a dependent position. It is used for clients who climb out of bed and are at risk for falling. Ambularm devices that may be attached to the bed or chair or to the client's mattress or nightgown are also available. A belt restraint is a device that is wrapped around the client's waist to secure the client to bed or to a stretcher. A wrist restraint is a device used to immobilize an arm. An elbow restraint consists of a piece of fabric with slots into which tongue blades are inserted, after which the device is wrapped around the elbow area to immobilize it. Of the options provided, the Ambularm is the least restrictive safety device.

An industrial nurse at a large factory provides information to the employees in the mailroom and shipping department about the signs of skin (cutaneous) anthrax. For which early sign of cutaneous anthrax does the nurse tell the employees to check their skin? A weeping blister A black skin area of skin An itchy bump An open ulcer

An itchy bump Rationale: Skin anthrax starts with an itchy bump (papule) that looks like a mosquito bite. It progresses to a small fluid-filled sac that becomes a painless ulcer with an area of dead black tissue in the middle. (Toxins from the anthrax spores destroy the surrounding tissue.)

A military nurse who is in charge of planning a vaccination clinic to administer the smallpox vaccine to military personnel is preparing a pamphlet that sets forth guidelines for care of the vaccination site. Which guideline should the nurse include in the pamphlet? Apply an antihistamine ointment to the scab to prevent itching. Keep the vaccination site open to air as much as possible. Avoid sharing towels or other items that have come in contact with the vaccination site. Soak the scab that forms with warm water every day.

Avoid sharing towels or other items that have come in contact with the vaccination site.

A community health nurse is providing an educational session on childhood poisoning at a local school. What does the nurse tell the group what the first action is when an accidental poisoning occurs? Induce vomiting Bring the child to the emergency department (ED) Call an ambulance Call the poison control center

Call the poison control center Rationale: When a poisoning occurs, a poison control center should be contacted immediately and any directions given regarding treatment followed. The poison control center will provide directions regarding the inducement of vomiting. However, vomiting should not be induced if the victim is unconscious or if the substance ingested is a strong corrosive or petroleum product. The poison control center may advise the mother to bring the child to the ED; if this is the case, the mother should call an ambulance. Neither bringing the child to the ED nor calling an ambulance would be the immediate actions, because either tactic would delay treatment.

Which of these interventions does a nurse manager, reviewing infection control interventions with the nursing staff, tell the staff will reduce reservoirs of infection? Select all that apply. Changing dressings that become wet or soiled Placing capped needles and syringes in puncture-resistant containers Using soap and water to remove drainage, dried secretions, or excess perspiration from a client's skin Emptying urinary drainage systems (Foley catheter drainage) on each shift unless prescribed otherwise by a physician Placing tissues and soiled dressings in paper bags Keeping bedside table surfaces clean and dry

Changing dressings that become wet or soiled Using soap and water to remove drainage, dried secretions, or excess perspiration from a client's skin Emptying urinary drainage systems (Foley catheter drainage) on each shift unless prescribed otherwise by a physician Keeping bedside table surfaces clean and dry Rationale: Measures to reduce reservoirs of infection include keeping bedside table surfaces clean and dry; placing tissues, soiled dressings, and soiled linens in moisture resistant bags (not paper bags); changing dressings that become wet or soiled; placing syringes and uncapped (not capped) needles in puncture-resistant containers; using soap and water to remove drainage, dried secretions, or excess perspiration from a client's skin; and emptying all drainage systems on each shift unless prescribed otherwise by a primary health care provider.

Which of the following points should the nurse include when documenting information about a client who is wearing wrist restraints? Select all that apply. Circulatory and neurovascular status of the restrained extremities The date and time of application of the restraint The procedure used in applying the restraint Skin integrity of the restrained body part The client's 24-hour urine output The client's temperature

Circulatory and neurovascular status of the restrained extremities The date and time of application of the restraint The procedure used in applying the restraint Skin integrity of the restrained body part Rationale: The nurse is responsible for documenting specific information about the client who is wearing any type of restraint. The points that must be included in such documentation are the reason for the restraint; alternatives to the restraint that were used; the method of restraint; the procedure used in applying the restraint; date and time of application of the restraint; client's response to application of the restraint; condition of the restrained body part; assessment of circulatory, neurovascular, and skin integrity; periodic release from restraint with movement or range-of-motion exercise; assessment of the need for continued use of the restraint; the duration of use of the restraint; and the client's response on removal of the restraint.

An adolescent client asks the nurse questions about the transmission of the Epstein-Barr virus (infectious mononucleosis). By which route should the nurse tell the client that the disease is transmitted? Respiratory droplets Close intimate contact Airborne particles Fecal-oral

Close intimate contact Rationale: Epstein-Barr virus is transmitted by way of contact with infectious saliva, close intimate contact with an infectious individual, or contact with infected blood. The infectious period is unknown, but the virus is commonly shed from before clinical onset of disease until 6 months or longer after recovery. It is not transmitted by way of the fecal-oral route, in airborne particles, or in respiratory droplets.

A nurse manager tells the nursing staff that the agency's disaster preparedness plan will be distributed to all employees for review. What does the nurse manager say is a primary component of disaster readiness? Identify the types of disasters that may occur Identify the location of healthcare supplies Describe a formal plan of action for the coordination of a response Aid determination of how victims will be triaged

Describe a formal plan of action for the coordination of a response Rationale: A primary component of a disaster preparedness plan is a formal plan of action for coordinating the response of a healthcare agency's staff. Depending on the agency, the disaster preparedness plan may be specific and may include other information such as the location of health care supplies, instructions for the triage of victims, and the types of disasters that may occur.

A nurse preparing to perform a sterile dressing change notes that the covering of a package of sterile 4 × 4 gauze pads has a small tear. Which action should the nurse take? Examine the gauze pads and using them as long as they appear untouched Discard the gauze pad closest to the outside of the package and using the others Discard the package Use the gauze pads, because the tear was small

Discard the package Rationale: When performing a surgically aseptic procedure, the nurse must follow certain principles of aseptic technique to ensure maintenance of asepsis. A sterile object remains sterile only when touched by other sterile objects. If the sterile state of an object is questionable (e.g., if there is a tear or break in the covering of a sterile object), the nurse must discard the object, because it is considered contaminated. Therefore the nurse in this situation would not use the gauze pads.

A nurse is providing instructions to a nursing assistant who will be caring for a client in hand restraints. How often does the nurse instruct the nursing assistant to release the restraints to permit muscle exercises? Every 30 minutes Every 3 hours Every 4 hours Every 2 hours

Every 2 hours Rationale: The nurse should instruct the nursing assistant to assess the restraints and the client's circulatory status and skin integrity every 30 minutes. Restraints must be released at least every 2 hours to permit muscle exercise and promote circulation. Agency guidelines regarding the use of restraints should always be followed.

A nurse educator is providing an in-service program to emergency department nurses about the signs/symptoms of inhalation anthrax. What does the nurse educator tell the nurses is an early indication of inhalation anthrax? Signs/symptoms of shock Respiratory distress Flulike signs/symptoms Hemorrhage

Flulike signs/symptoms Rationale: Inhalation anthrax is caused by the inhalation of spores from Bacillus anthracis, which multiply in the alveoli. This form of anthrax begins with the same signs/symptoms as the flu, including fever, muscle aches, and fatigue. Signs/symptoms suddenly become more severe with the development of breathing problems and shock. Toxins from the anthrax spores cause hemorrhage and destruction of lung tissue.

A registered nurse(RN)is watching as a new licensed practical nurse(LPN) suctions a client with a diagnosis of acquired immunodeficiency syndrome (AIDS). Which of the following protective devices worn by the LPN would cause the RN to determine that the LPN was performing the procedure safely? Gloves, gown, and face shield Gown and protective eyewear Gloves and mask Gloves and gown

Gloves, gown, and face shield Rationale: The RN is responsible for supervising procedures performed by a new LPN to ensure that safety is maintained and that policies and procedural guidelines are followed. Standard precautions include use of gloves whenever there will be actual contact with blood or body fluids or the potential for contact exists. Therefore the LPN must wear gloves. The LPN also needs to protect the eyes, nose, and mouth from contact with the client's respiratory secretions; a face shield will provide this protection. A mask or protective eyewear does not provide adequate protection. Gowns are worn in those instances when it is anticipated that there will be contact with body fluid or blood.

A nurse is attending an in-service program on disaster preparedness. Which of the following events are described as examples of natural disasters? Select all that apply. Hurricane Terrorist attack Toxic waste spill Drought Bus accident Flood

Hurricane Drought Flood Rationale: A disaster is any human-made or natural event that results in destruction and devastation that cannot be alleviated without assistance (i.e., medical, local, or federal government assistance). A natural disaster usually cannot be prevented, whereas a human-made disaster can be prevented. Flood, drought, and hurricane are the only natural disasters identified in the options. Bus accidents, terrorist attacks, and toxic waste spills are all human-made disasters.

The nursing staff in an emergency department is reviewing and updating the disaster preparedness plan. The staff members, discussing ways to help prevent the transmission of smallpox, know that this infection is transmitted by which route? Through open wounds Enteric Inhalation Direct contact Gastrointestinal Handling of contaminated materials

Inhalation Direct contact Handling of contaminated materials Rationale: Smallpox, transmitted in air droplets, direct contact and in the handling of contaminated materials, is highly contagious. Signs/symptoms include fever, back pain, vomiting, malaise, and headache, followed 2 days later by the appearance of papules that progress to pustular vesicles, which are initially abundant on the face and extremities. Enteric, gastrointestinal, and open wounds are not routes of smallpox transmission.

The nurse plans to wear this protective mask (see figure) when caring for clients with certain disorders. What are these disorders? Select all that apply. Meningococcal pneumonia Hepatitis A Pharyngeal diphtheria Streptococcal pharyngitis Tuberculosis Scabies

Meningococcal pneumonia Pharyngeal diphtheria Streptococcal pharyngitis Rationale: A standard mask is used as part of droplet precautions to protect the nurse from acquiring the client's infection. Droplet precautions are those precautions used to help prevent the spread of organisms that can spread through the air but are unable to remain in the air farther than 3 feet (1 meter) from the source. Many respiratory viral infections require the use of a standard mask during client care. Some of the disorders requiring the use of a standard mask are pharyngeal diphtheria; rubella; streptococcal pharyngitis; pertussis; mumps; pneumonia, including meningococcal pneumonia; and pneumonic plague. Scabies and hepatitis A, transmitted by way of direct contact with an infected person, require the use of contact precautions for protection. Tuberculosis requires airborne precautions and the use of an individually fitted particulate filter mask. A standard mask would not protect the nurse from Mycobacterium tuberculosis.

A nurse is questioning a client about hazards in the home environment. Which of the following items in the home is an indication that the client requires instruction about safety? Select all that apply. Clothes hamper at the end of the hallway Small rugs in the living room Untacked rugs on the stairs Cereal boxes, canned foods, and infrequently used cooking utensils stored on top of the refrigerator Carpet on stairs secured with tacks

Small rugs in the living room Untacked rugs on the stairs Cereal boxes, canned foods, and infrequently used cooking utensils stored on top of the refrigerator Rationale: Area rugs and runners should not be used on or near stairs. Injuries in the home are frequently the result of loose objects, including small rugs on the stairs or floor, wet spots on the floor, or clutter on bedside tables, closet shelves, the top of the refrigerator, and bookshelves. Care should also be taken to ensure that end tables are secure and have stable straight legs. Nonessential items should be placed in drawers to eliminate clutter. If the stairs must be carpeted, carpeting should be secured with the use of tacks.

A community health nurse is asked to assist in developing a community disaster plan identified by Federal Emergency Management Agency (FEMA). The nurse knows that the preparedness phase of the plan includes what components? Select all that apply. Training of disaster personnel Planning for rescue Caring for disaster victims Putting disaster planning services into action Evacuation Actions to prevent the occurrence of a disaster or reduce the damaging effects

Training of disaster personnel Planning for rescue Caring for disaster victims Evacuation Rationale: The preparedness phase has many functions, including planning for rescue, evacuation, and caring for disaster victims; the training of disaster personnel and gathering of resources, equipment, and other materials needed in dealing with a disaster; identifying specific responsibilities for various disaster response personnel; establishing a community disaster plan and an effective public communication system; setting up an emergency medical system and a plan for its activation; checking for proper function of emergency equipment; making anticipatory provisions and setting up a location for food, water, clothing, medication, shelter, and other supplies; checking supplies on a regular basis and replenishing outdated materials; and practicing community disaster plans (mock-disaster drills). The mitigation phase refers to actions or measures to either prevent the occurrence of a disaster or reduce the damaging effects of a disaster. The response phase includes putting disaster planning services into action and the actions taken to save lives and prevent further damage. The recovery phase includes actions taken to return to a normal situation after the disaster.

A home care nurse visits a client who lives in a small apartment to perform an admission assessment. During the home safety assessment, the client asks the nurse whether it is safe to use a space heater. What is the appropriate response by the nurse? "A space heater should never be used in an apartment." "A space heater can be used as long as it is kept in the bedroom at night in case a fire occurs." "A space heater can be used as long as it is kept at a low setting at all times." "A space heater can be used as long as it's placed at least 3 feet (1 meter) from anything that may ignite."

"A space heater can be used as long as it's placed at least 3 feet (1 meter) from anything that may ignite." Rationale: Space heaters must be used appropriately because of the risk of fire. A space heater should be placed at least 3 feet (1 meter) from anything that may ignite. A space heater may be used in an apartment if there is ample space and safety precautions are followed. A low setting does not reduce the risk of fire. Placing a heater in a bedroom does not guarantee that it will be 3 feet (1 meter) from anything that may ignite.

A post office employee with suspected skin anthrax asks the emergency department nurse whether the infection is curable. What is the appropriate response by the nurse? "That's hard to say. We won't know for a week or two." "You really need to ask your primary health care provider." "Antibiotic therapy is usually prescribed and will cure the infection." "It is not curable, but fortunately, unlike inhalation anthrax, it is not deadly."

"Antibiotic therapy is usually prescribed and will cure the infection." Rationale: Skin anthrax starts with an itchy bump (papule) that looks like a mosquito bite. It progresses to a small fluid-filled sac that becomes a painless ulcer with an area of dead black tissue in the middle. (Toxins from the anthrax spores destroy surrounding tissue.) Antibiotic treatment cures this infection, but untreated skin anthrax can result in overwhelming septicemia and death. Replying, "You really need to ask your primary health care provider about that" or "That's hard to say. We won't know for a week or two" is nontherapeutic and places the client's question on hold. Stating, "It is not curable, but fortunately, unlike inhalation anthrax, it is not deadly" is incorrect.

A nurse prepares to teach a client with chronic vertigo about safety measures to help prevent exacerbation of signs/symptoms and injury. Which instructions should the nurse provide to the client? Select all that apply. "Use public transportation as much as possible." "Change positions slowly." "Remove clutter from your home." "Drive your car only if you're not feeling dizzy." "Turn your head slowly when someone speaks to you."

"Change positions slowly." "Remove clutter from your home." Rationale: Any sudden movement could precipitate a vertigo attack, so, to help prevent vertigo attacks, the client should avoid such movements. The client with chronic vertigo should avoid driving; the use of public transportation should also be avoided because of the sudden movements that occur with this mode of transport. The client should also change position slowly and should turn the entire body, not just the head, when spoken to. If vertigo does occur, the client should immediately sit down or grasp the nearest piece of stable furniture. The client should maintain the home in a state free of clutter and remove throw rugs, because the effort of trying to regain balance after slipping could trigger the onset of vertigo.

The unit supervisor of an emergency department (ED) is called at home and told by an emergency department nurse who is on duty that an airplane crash has occurred and numerous casualties will be arriving at the ED. What should the initial response by the unit supervisor be? "Has the disaster plan been activated?" "Be sure that the nursing staff finds as many stretchers as they can." "Call as many nursing staff as you can to come in to work." "Make sure all of the rooms are well stocked with supplies."

"Has the disaster plan been activated?" Rationale: In an external disaster, many people will be brought to the ED for treatment. Although ensuring that rooms are well stocked with supplies, calling nursing staff to come to work, and finding stretchers are components of preparing for the casualties, the initial nursing action must be activation of the disaster plan. Therefore the initial response by the unit supervisor should be "Has the disaster plan been activated?"

A nurse, assessing a client's readiness for discharge, is performing a home safety assessment to determine whether there are any environmental hazards in the home. Which of the following statements, if made by the client, would prompt the nurse to investigate further? Select all that apply. "I keep my personal items within reach when I sit in my easy chair." "I've removed the scatter rugs from the house." "I haven't changed the batteries in the smoke detectors in my home for quite a few years now." "I live in a single-story house." "I don't have any nightlights in the house."

"I haven't changed the batteries in the smoke detectors in my home for quite a few years now." "I don't have any nightlights in the house." Rationale: If the client tells the nurse that there are no nightlights in the home, the nurse should further investigate the situation. Nightlights help prevent falls by the client who may need to get up during the night. The batteries in smoke detectors should be changed at least once a year, so the nurse must investigate further if the client indicates that this has not been done for "quite a few years." The other statements by the client do not reflect environmental hazards in the home.

A home health nurse has instructed a client about safety measures during the use of an oxygen concentrator in the home. Which statement by the client indicates to the nurse that the client understands the directions? Select all that apply. "I need to keep the oxygen concentrator as close to the wall as possible or put it in a corner." "I need to follow the oxygen prescription exactly." "I have to tell everyone that they can't smoke or have an open flame within 10 feet (3 meters) of the oxygen concentrator." "I can use my electric razor while I'm using oxygen." "I have to keep the oxygen concentrator out of direct sunlight."

"I need to follow the oxygen prescription exactly." "I have to tell everyone that they can't smoke or have an open flame within 10 feet (3 meters) of the oxygen concentrator." "I have to keep the oxygen concentrator out of direct sunlight." Rationale: The client should follow the oxygen prescription exactly. The use of electric razors or other equipment that could emit sparks should be avoided while oxygen is in use, because fire and injury to the client could result. The oxygen concentrator is kept out of direct sunlight and slightly away from walls and corners to permit adequate air flow. The client should not allow smoking or any type of flame within 10 feet (3 meters) of the oxygen source. Other measures include having telephone numbers for the primary health care provider, nurse, and oxygen vendor available and teaching the client signs/symptoms requiring emergency care.

A home health nurse teaches a client about home modifications to reduce the risk of falls. Which statements by the client indicate a need for further teaching? Select all that apply. "I need to use night lights." "I need to use the staircase handrails when I go up the stairs." "I need to remove my wall-to-wall carpeting." "I should walk barefoot as much as possible so that I'll know about any wet spots on the floor." "I need to get handrails put up in the bathroom."

"I need to remove my wall-to-wall carpeting." "I should walk barefoot as much as possible so that I'll know about any wet spots on the floor." Rationale: Home modifications to reduce the risk of falls include ensuring ample lighting, removing scatter rugs, placing handrails in bathrooms, and using handrails on all staircases. The client should wear flat rubber-soled shoes to prevent slips and falls. Walking barefoot will not reduce the risk of injury; in fact, it could actually increase the risk of foot injury and of slipping and falling. Removal of wall-to-wall carpeting is not necessary.

A client with osteoporosis is at risk for falls. Which statement by the client indicates the need for instruction regarding measures to prevent falls? "I use a shower chair when I bathe." "The railings on my stairs are sturdy and secure." "I've placed nightlights in my hallway." "I took the bathmat out of my tub."

"I took the bathmat out of my tub." Rationale: Home modifications to reduce the risk for falls include use of sturdy, secure railings on all staircases and ample lighting, including nightlights. Bathroom safety equipment includes a shower chair, handrails in the shower and near the toilet, and a mat in the tub to prevent slipping.

A nurse has provided instructions to a mother regarding the use of safety seats in car travel for her newborn infant. Which statement by the mother indicates understanding of the instructions? "I'll put the baby's car seat in the middle back seat, facing backward and reclined a little." "I'll put the baby's car seat in the middle back seat, facing forward and reclined a little." "I'll put the baby's car seat in the front seat, facing forward and reclined a little." "I'll put the baby's car seat in the front seat, facing backward and reclined a little."

"I'll put the baby's car seat in the middle back seat, facing backward and reclined a little." Rationale: The infant should be restrained in a car seat in a semireclined, rear-facing position to allow the seat and infant's spine to bear the forces of impact should a collision occur. The infant should never face forward or ride in the front seat.

A teenage client returns to the gynecological (GYN) clinic for a follow-up visit after diagnosis and initial treatment of a sexually transmitted infection (STI). Which statement by the client indicates the need for further teaching? "I know you won't tell my parents that I'm sick." "I always make sure my boyfriend uses a condom." "My boyfriend doesn't have to come in for treatment." "I finished all the antibiotic, just like you said."

"My boyfriend doesn't have to come in for treatment." Rationale: In the treatment of STIs, all sexual contacts must be alerted and treated with medication. Any treatment at a GYN clinic for teenagers is confidential, and parents will not be contacted even if the client is under 18 years. The client should always finish the medication prescribed by the primary healthcare provider. Every client who is being treated for an STI or is at risk for an STI should use a condom for any sexual contact

A fever develops in a client who has been hospitalized for 2 months and is receiving parenteral nutrition by way of a central venous line, and central venous line-related sepsis is diagnosed. What does the nurse interpret the meaning of this infection? Select all that apply. A hospital-acquired infection A healthcare-associated infection A nosocomial infection A community-acquired infection An iatrogenic infection A result of bacterial colonization

A hospital-acquired infection A healthcare-associated infection A nosocomial infection Rationale: Infections that occur during hospitalization, or are a result of hospitalization, are referred to as healthcare-associated infections, hospital-acquired infections, or nosocomial infections. Colonization is defined as a condition in which microorganisms are present in body tissues; there is no damage to the tissues, and no local signs/symptoms of infection are evident. Iatrogenic infections are infections that involve the client's normal flora. A community-acquired infection is an infection that the person is admitted with or is incubating on admission to the hospital.

An emergency department (ED) nurse is triaging victims of an explosion at a nearby manufacturing plant. To which victims should the nurse assign the emergent (priority 1) designation? Select all that apply. A victim who is dazed and staggering around the other victims A victim with burns of both arms A victim who is alert but complaining of loss of vision A victim with a limb amputation A victim who has sustained minor bruising of an arm and the lower legs

A victim who is dazed and staggering around the other victims A victim with a limb amputation Rationale: The emergent designation (a.k.a. red or priority 1), the highest priority, is assigned to the victim who has sustained life-threatening injuries and requires immediate attention and continuous evaluation yet has a high probability of survival once his or her condition has been stabilized. In this scenario, emergent status should be assigned to the victim with a limb amputation, the victim with vision loss, and the victim who is bleeding profusely. The victim who is dazed and staggering around the other victims may also be assigned to this category because it is possible that the client has sustained a head injury. The urgent designation (a.k.a. yellow or priority 2) is given to the victim who requires treatment but whose injuries and their complications are not life threatening, provided that they are treated within 1 to 2 hours. The nonurgent (a.k.a. green or priority 3) designation is given to the victim with local injuries who does not exhibit immediate complications and who will be able to wait several hours for medical treatment; such victims require evaluation every 1 to 2 hours thereafter. In this scenario, the victim who has sustained minor bruises of the arm and lower legs would be assigned to this category.

A nurse responds to an external disaster that occurred in a large city when a building collapsed. Numerous victims require treatment. Which victim should the nurse attend to first? An alert victim who has numerous bruises on the arms and legs A victim with a partial amputation of a leg who is bleeding profusely A hysterical victim who has sustained a head injury A victim who has died of multiple serious injuries

A victim with a partial amputation of a leg who is bleeding profusely Rationale: The nurse determines which victim will be attended to first on the basis of the severity of injury of each of the victims of the disaster. An injury that threatens life, limb, or vision without immediate attention is categorized as emergent and is the priority (in this case, the victim with a partial amputation who is bleeding profusely). A victim who requires treatment but whose life, limbs, and vision are not threatened if care can be provided within 1 to 2 hours is considered to represent an urgent case and is the second priority (here, the hysterical victim who has sustained a head injury). Local injuries that require evaluation and possibly treatment but for which time is not critical are categorized as nonurgent and represent the third priority (here, the victim with numerous bruises on the arms and legs). Caring for a victim who is already dead is the final priority.

A triage nurse in an emergency department (ED) is attending to the victims of a train crash. All victims are alert. Which of these clients does the nurse assign to the emergent category? Select all that apply. A victim with a fractured humerus A victim with respiratory distress A victim with a forehead laceration that is not bleeding A victim with multiple nonbleeding bruises of the arms and legs A victim with partial amputation of the foot

A victim with respiratory distress A victim with partial amputation of the foot Rationale: One rating system commonly used in the ED consists of three tiers — emergent, urgent, and nonurgent — with the categories sometimes identified with color coding or numbers. The emergent classification (a.k.a. red or priority 1) is given to clients with life-threatening injuries (here, the clients with respiratory distress [airway] and partial amputation of the foot [bleeding/circulation]) who require immediate attention and continuous evaluation but have a high chance of survival once their conditions have been stabilized. The urgent (a.k.a. yellow or priority 2) classification is given to clients whose injuries and complications are not life threatening (here, the client with the fractured humerus), provided that they are treated within 1 to 2 hours; such clients require evaluation every 30 to 60 minutes thereafter. The nonurgent (a.k.a. green or priority 3) classification is given to clients with local injuries (here, the clients with the forehead laceration and bruises of the arms and legs) who do not have immediate complications and can wait several hours for medical treatment; these clients require evaluation every 1 to 2 hours thereafter.

A hurricane is forecast to make landfall in 48 hours, and the staff of the emergency department of an area hospital is advised to prepare for causalities. Which action should the nurse manager who receives the telephone call regarding this warning take first? Increase the number of nursing staff for the day on which the hurricane is expected Supply the triage rooms with additional equipment Activate the agency disaster plan Call the hospital maintenance department to secure the building against the storm

Activate the agency disaster plan Rationale: In an external disaster, many people may be brought to the emergency department for treatment. Although increasing the nursing staff and supplying the triage rooms with additional equipment may be steps in preparing for casualties, the initial action by the nurse manager must be activation of the disaster plan. Calling the hospital maintenance department to secure the building from the storm is not a responsibility that falls within the scope of nursing management.

A nurse in a post-anesthesia care unit (PACU) receives a client from the operating room. For what finding should the PACU nurse assess the client first? Airway patency Active bowel sounds Orientation to surroundings Adequate urine output

Airway patency Rationale: After a client's transfer from the operating room, the PACU nurse performs an assessment, assessing airway patency first. The client may not have active bowel sounds at this time as a result of the effects of anesthesia. Urine output and orientation to surroundings may also be assessed, but these are not the first priorities.

An older client is extremely anxious after admission, having never been hospitalized before. To help provide a safe environment and minimize the stress of hospitalization on the client, what does the nurse plan to do? Select all that apply. Keep visitors to a minimum Allow the client to have as many choices regarding his care as possible Acknowledge the client's feelings Keep the door open and the room lights on at all times Put the client in a room far from the nurses' station Provide information about hospital routines

Allow the client to have as many choices regarding his care as possible Acknowledge the client's feelings Provide information about hospital routines Rationale: Several general interventions can be used to minimize stress in the hospitalized client. These include acknowledging the client's feelings, providing information, providing social support, and giving the client control, when possible, over choices related to care. Admitting the client to a room far from the nurses' station and limiting visitors would both serve to increase the client's anxiety. Keeping the door open and the room lights on at all times could cause further disruption in the client's sleep pattern in addition to the disruption created by the hospitalization.

A nurse is performing sterile wound irrigation for an assigned client. A nursing assistant enters the client's room and tells the nurse that a primary health care provider has telephoned and has asked to speak to the nurse. What is the appropriate action by the nurse? Asking the nursing assistant to take a message Covering the client and answering the telephone call Finishing the wound irrigation while the primary health care provider waits on the telephone Asking the nursing assistant to obtain a telephone number from the primary health care provider so that the nurse may return the call after the wound irrigation is complete

Asking the nursing assistant to obtain a telephone number from the primary health care provider so that the nurse may return the call after the wound irrigation is complete Rationale: Because wound irrigation is a sterile procedure and a risk for infection exists with any open wound, the nurse should ask the nursing assistant to obtain a telephone number from the primary health care provider so that the call may be returned after the wound irrigation is complete. It is not appropriate to ask a primary health care provider to wait while a procedure is being completed. It is best to return the call. It is not the responsibility of the nursing assistant to take a message.

A licensed practical nurse (LPN) tells the registered nurse (RN) that she administered acetaminophen to a client by way of the rectal route rather than the prescribed oral route because the client was extremely nauseated. What is the most appropriate action by the RN? Tells the LPN that she made a sound judgment in administering the medication by way of the rectal route Instructs the LPN to write "pr" (per rectum) on the medication record next to the time at which the medication was administered Asks the LPN to check the client in 30 minutes to see whether the nausea has subsided Asks the LPN to complete and file an incident report

Asks the LPN to complete and file an incident report Rationale: If a medication is prescribed to be administered by way of the oral route, the nurse may not use an alternate route to administer the medication unless the change is prescribed by the primary health care provider. The nurse would ask the LPN to complete and file an incident report because the LPN, legally speaking, made a medication error. Telling the LPN that she made a sound judgment in administering the medication by way of the rectal route is incorrect. Although the client must be reassessed and the LPN would document administration of the medication by way of the rectal route in the client's record, the most appropriate option given is having the LPN complete and file an incident report.

A physician writes a prescription for the application of a heating pad to a client's back. Which of the following actions should the nurse take when implementing this prescription? Select all that apply. Assessing the client's medical history and risk factors for burns Adjusting the heating pad to the high setting Assessing the heating pad periodically for proper electrical function Placing the heating pad under the client Frequently assessing the client's skin for signs of burns

Assessing the client's medical history and risk factors for burns Assessing the heating pad periodically for proper electrical function Frequently assessing the client's skin for signs of burns Rationale: The nurse should first assess the client's medical history, including risk factors for burns. The heating pad should never be placed under a client; instead, it should be placed lightly against or on top of the involved area. Burns may result when a client lies on a heating pad. The heating pad is adjusted to the low setting; the high setting can cause burns. Assessing the client for altered skin integrity and checking for proper electrical function are appropriate measures for the use of a heating pad.

A military nurse who is in charge of planning a vaccination clinic to administer the smallpox vaccine to military personnel is preparing a pamphlet that sets forth guidelines for care of the vaccination site. Which guideline should the nurse include in the pamphlet? Apply an antihistamine ointment to the scab to prevent itching. Keep the vaccination site open to air as much as possible. Avoid sharing towels or other items that have come in contact with the vaccination site. Soak the scab that forms with warm water every day.

Avoid sharing towels or other items that have come in contact with the vaccination site. Rationale: A scab will form in the spot where the vaccination was administered. This scab should be left alone so that the vaccinia virus in the vaccine doesn't spread to other parts of the body. The site is loosely covered with a gauze bandage; this bandage, in turn, is covered with a waterproof bandage during bathing. Clothing is worn over the vaccination site as an extra precaution. The hands must be washed frequently, including whenever the site is touched or the bandage is changed; the vaccine should not touch the eyes or any other part of the body after changing the bandage or touching the vaccination site. The vaccine must avoid scratching or putting ointment on the vaccination site. The vaccine is told to avoid sharing towels and to launder items that have touched the vaccination site because of the risk of spread of the vaccinia virus.

A sedated client is being transported to the radiology department on a stretcher. Which type of restraint should the nurse suggest applying to help ensure the client's safety? Wrist Mitten Elbow Belt

Belt Rationale: A belt restraint is a device that is wrapped around the client's waist to secure the client to bed or to a stretcher. An elbow restraint consists of a piece of fabric with slots into which tongue blades are inserted; the device is wrapped around the elbow area to keep it immobile. A mitten restraint is a thumbless device that covers the client's hand and is used to restrain the client's hand, preventing the client from dislodging invasive equipment (e.g., an intravenous line). A wrist restraint is a device used to immobilize an arm that does not allow movement as a mitten restraint would.

The mother of a 3-year-old calls a neighbor who is a nurse and reports that her child just drank some window cleaner that had been stored in a cabinet. What should the nurse instruct the mother to do immediately? Administer an excessive amount of fluids to induce vomiting Call a poison control center Leave a message on the primary health care provider's answering service about the incident Call an ambulance to bring the child to the emergency department

Call a poison control center Rationale: When a poisoning occurs, a poison center should be called immediately. Vomiting should not be induced if the victim is unconscious or if the substance ingested was a strong corrosive or petroleum product. Also, vomiting should not be induced unless a primary healthcare provider has given specific instructions to induce vomiting. Neither calling an ambulance nor calling the primary health care provider's answering service is the immediate action because either would delay treatment. Additionally, the primary health care provider would immediately make a referral to the poison control center. The poison control center may advise the mother to bring the child to the emergency department; if this is the case, the mother should then call an ambulance.

A nurse manager of an emergency department (ED) arrives at work and is told that four registered nurses scheduled to work will not be reporting to work because they are ill. Every trauma room is busy, and emergency medical services (EMS) has just called to report that several victims involved in a 10-car wreck on the interstate will be brought to the ED. How does the nurse manager initially manage this situation? Calling the nursing supervisor to discuss activation of the disaster plan Closing the emergency department temporarily to incoming clients Demanding that the nurses from the night shift stay until all of the victims have been treated Telling EMS to take the victims to another hospital

Calling the nursing supervisor to discuss activation of the disaster plan Rationale: External disasters occur in the community, and many victims of such events are brought to the ED for care. In this situation, the nurse manager would initially call the nursing supervisor to discuss the need for additional staffing and activation of the disaster plan. The nurse manager would not ask EMS to take the victims to another hospital or temporarily close the ED to incoming clients; such decisions are made by hospital administrators. The nurse manager should ask, not demand, that nurses from the night shift stay until all of the victims have been treated.

A nurse in a long-term care facility recognizes the need to place wrist restraints on a client, but the client does not want the restraints applied. What would be the most appropriate action by the nurse? Medicate the client with a sedative, then apply the restraints Apply the restraints anyway Contact the primary health care provider Compromise with the client and use only one wrist restraint instead of two

Contact the primary health care provider Rationale: The use of restraints must be avoided if possible. If it is determined that a restraint is necessary, the nurse should discuss the issue with the family and obtain a prescription from the primary health care provider. The nurse should explain carefully to the client and family the reasons that the restraint is necessary, the type of restraint that has been selected, and the anticipated duration of use of the restraint. If a client refuses restraints, the nurse must contact the primary health care provider. Therefore, the other options are incorrect.

A nurse is discussing accident prevention with the family of an older client who is being discharged from the hospital after hip surgery. This client has a tendency to be forgetful. Which items in the home increase the client's risk for injury? Select all that apply. A water heater thermostat adjusted to a low setting Cooking equipment such as a stove A nightlight in the bathroom Smoke and carbon monoxide detectors Common household objects such as door mats Elevated toilet seat with armrests

Cooking equipment such as a stove Common household objects such as door mats Rationale: Physical hazards in the environment place the client at risk for accidental injury and death. Adequate lighting, such as nightlights in dark hallways and bathrooms, reduces the physical hazard by illuminating areas in which a person moves about. An elevated toilet seat with armrests and nonslip strips on the floor in front of the toilet are useful in reducing the incidence of falls in the bathroom. Cooking equipment and appliances, particularly stoves, are a major cause of fires and related injuries in the home, especially if the client is forgetful and leaves the stove or oven on after cooking. Smoke and carbon monoxide detectors should be placed throughout the home to alert members of the household to danger. A low thermostat setting on the water heater reduces the risk of burns during the use of hot water (e.g., bathing or showering). Injuries in the home are often the result of tripping over or coming into contact with such common household objects as a door mats, small rugs on the floor or stairs, and clutter around the house.

A nurse is preparing to initiate a continuous tube feeding, using a tube-feeding pump. On bringing the pump to the bedside and preparing to plug the pump in, the nurse discovers that there is no available plug in the wall socket. What is the most appropriate action the nurse should do? Ask the primary health care provider to change the prescription to intermittent feedings Plug in the pump cord into an available plug above the sink Determine the need for the appliances now plugged into the needed wall socket Use a regular extension cord to allow the use of more than one electrical appliance

Determine the need for the appliances now plugged into the needed wall socket Rationale: It is most appropriate for the nurse to assess the situation and determine the need for the appliances already plugged into the needed wall socket.The use of electrical appliances near a sink presents a hazard. It is not appropriate (and is premature) to ask the primary health care provider to change the prescription, because the prescription is based on the client's needs. A regular extension cord should not be used, because it poses a risk of fire.

According to the Federal Emergency Management Agency (FEMA) description of the phases of disaster management, what are the components of actions in the mitigation phase? Select all that apply. Determining available resources for the care of infants, older clients, the disabled, and people with chronic health problems Actions that can prevent the occurrence of a disaster Measures that can reduce a disaster's damaging effects Putting disaster-planning services into action Actions taken to return to normal after the disaster Actions that plan for rescue, evacuation, and care of disaster victims

Determining available resources for the care of infants, older clients, the disabled, and people with chronic health problems Actions that can prevent the occurrence of a disaster Measures that can reduce a disaster's damaging effects Rationale: The mitigation phase consists of actions or measures that can either prevent the occurrence of a disaster or reduce a disaster's damaging effects. The task of determining the resources available for the care of infants, older clients, the disabled, and people with chronic health problems is addressed in this phase. The preparedness phase involves actions that plan for rescue, evacuation, and care of disaster victims. The response phase involves putting disaster-planning services into action and enumerating the actions needed to save lives and prevent further damage. The recovery phase includes actions taken to return to normal after the disaster.

A community health nurse working in a school setting is concerned because parents are not participating in health activities designed to promote child safety. What is the most appropriate initial action for the nurse to take? Implementing a child safety program Determining the appropriateness of the planned health activity Performing an analysis of health problems related to child safety Planning a focused child safety program

Determining the appropriateness of the planned health activity Rationale: In this situation, the most appropriate initial action would be to determine the appropriateness of the planned health activities. This would be followed by analysis, planning, and implementation.

A home health nurse is visiting a client with tuberculosis (TB). Which action by the client tells the nurse that the client understands the necessary respiratory precautions to be taken at home? Wearing an oxygen mask at all times Staying secluded in the bedroom Disposing of contaminated tissues in a container with a leak-proof bag Keeping the house closed up to minimize the spread of disease

Disposing of contaminated tissues in a container with a leak-proof bag Rationale: The client under respiratory precautions at home does not need to remain secluded; the client would not be at home if he/she were infectious. However, proper respiratory precautions are necessary. The house should be properly ventilated, and the windows should be opened as much as possible. Wearing an oxygen mask at all times is not a respiratory precaution, and there is no information in the question to indicate that oxygen is necessary. Contaminated tissues should be discarded in a container with a leak-proof bag and then placed in an outdoor trash bin. Tissues should not be left lying around.

A nurse is preparing a sterile field to change a client's sterile dressing. What are some sterile techniques the nurse must adhere to? Select all that apply. Cuff the top of the disposable paper bag, and place it within reach of the work area. Do not turn your back to the sterile field at any time. A half-inch border of the sterile field is considered contaminated. Make sure to use sterile gloves when opening up sterile gauze packages to place on the sterile field. Maintain the sterile field and gloved hands above the level of the waist.

Do not turn your back to the sterile field at any time. Maintain the sterile field and gloved hands above the level of the waist. Rationale: Remember, the nurse cannot turn his or her back to the field at any time, and with sterile technique, the sterile field and gloved hands must be maintained above the level of the waist. The 1-inch (2.5 cm), not half-inch, border on the field is considered contaminated. Also, the disposable bag must be a waterproof bag, not a paper bag. In addition, it is not necessary to use sterile gloves when opening up sterile gauze packages to place on the sterile field.

A nurse is assigned to care for a client with an infection caused by methicillin-resistant Staphylococcus aureus (MRSA). The client has an abdominal wound that requires irrigation and has a tracheostomy attached to a mechanical ventilator that requires frequent suctioning. While gathering the needed supplies before entering the client's room, which necessary protective items does the nurse obtain? Select all that apply. Gown Face shield Gloves Mask Shoe protectors

Gown Face shield Gloves Rationale: Infection caused by MRSA necessitates contact precautions. The care of this client requires the use of gown, gloves, and a face shield. The face shield is worn to protect the face and the mucous membranes of the mouth, nose, and eyes during interventions that could produce splashes of blood, body fluids, secretions, and excretions (e.g., wound irrigation and suctioning). Contact precautions also require the use of gloves and a gown if direct client contact is anticipated. A mask does not provide adequate protection. Shoe protectors are not necessary.

A nurse is preparing a disaster preparedness checklist, identifying emergency plans and supplies that will be needed in the event of a disaster, for a community group. Which instructions should be included on the list? Select all that apply. Have a firearm or other weapon available. Have an adequate supply of prescription medications. Obtain a 1-day supply of water (1 gallon per person). Plan a meeting place for family members. Have a first aid kit available. Have a battery-operated radio and a flashlight and batteries available.

Have an adequate supply of prescription medications. Plan a meeting place for family members. Have a first aid kit available. Have a battery-operated radio and a flashlight and batteries available. Rationale: Personal preparedness for a disaster includes planning a meeting place for family members, identifying safe spots in the home for each type of disaster; having a 3-day supply of water (1 gallon per person per day) and a 3-day supply of nonperishable food; and having clothing and blankets, a first aid kit, a battery-operated radio, a flashlight, and batteries available. For safety reasons, the nurse would not recommend that a weapon be kept.

A nurse is reading an article about the role of the American Red Cross (ARC) in a disaster. Which of the following responsibilities does the article ascribe to the ARC? Select all that apply. Identifying and training personnel for disaster response Developing a federal disaster response plan Educating the public about ways to prepare for disasters Declaring a disaster Providing disaster relief Activating disaster medical assistant teams

Identifying and training personnel for disaster response Educating the public about ways to prepare for disasters Providing disaster relief Rationale: The ARC has been given authority by the federal government to provide disaster relief. This organization works with the government in developing and testing community disaster plans, identifying and training personnel for disaster response, working with businesses and labor organizations to identify resources and people for disaster work, and educating the public about ways to prepare for disasters. Other responsibilities include operating shelters, providing assistance to meet immediate emergency needs, and providing disaster health services. Declaring a disaster, developing a federal disaster response plan, and activating disaster medical assistant teams are responsibilities of the Federal Emergency Management Agency.

An older client in a long-term care facility is at risk for injury because of confusion. Which of the following devices would be the best choice to help prevent injury while the client is in bed? Image of Mitt restraints Image with a device hanging from what appears to be the end of the bed Image of wrist restraints Image of patient sitting in bed with restraint wrapped around waist

It is the image with a device hanging from what appears to be the end of the bed. (Other options include mittens, wrist restraints and belt restraint - wrapped around patient's waist with straps across bed) Rationale: If the client is confused, the least intrusive method of restraint is the use of a bed alarm such as the Bed-Check bed exit alarm device. It consists of a weight-sensitive mat, placed on the client's mattress, that sounds an alarm when the sensor detects the removal of pressure. A belt restraint secures the client to the bed or stretcher. It restrains the center of gravity and prevents the client from sitting up on or rolling off a stretcher or falling out of bed. The extremity (ankle or wrist) restraint is used to immobilize an extremity as a means of protecting the client from injury resulting from a fall or the accidental removal of a therapeutic device such as a Foley catheter. The mitten restraint is a thumb less mitten device that is used to restrain the client's hand. It prevents the client from dislodging invasive equipment, removing dressings, or scratching him- or herself.

Which of the following statements reflect the principles of sterile technique? Select all that apply. Items in a sterile package must be used immediately once the package has been opened; otherwise they are considered contaminated. Sterile objects that come in contact with unsterile objects are to be considered contaminated. The clients overbed table is wiped with chlorhexidine. When a sterile field becomes wet, it remains sterile as long as the items on the field are not touched. If a package is not labeled as sterile, it should be considered unsterile. Any part of a sterile field that hangs below the top of the table is sterile as long as it is not touched.

Items in a sterile package must be used immediately once the package has been opened; otherwise they are considered contaminated. Sterile objects that come in contact with unsterile objects are to be considered contaminated. If a package is not labeled as sterile, it should be considered unsterile. Rationale: The term sterile means the absence of all microorganisms. To maintain sterile technique, the nurse must follow several principles. Among these principles: The edge of a sterile field and 1 inch (2.5 cm) inward is unsterile; sterile packages are labeled as sterile and, if the package is not so labeled, it is considered unsterile; sterile objects that come in contact with unsterile objects are considered contaminated; any part of a sterile field that falls or hangs below the top of the table is unsterile; cleaning an overbed table with chlorhexidine in not sterile; a sterile field that becomes wet will draw microorganisms from the surface beneath, contaminating the field; and items in a sterile package must be used immediately once the package has been opened, or they will be considered contaminated.

What is an example of the nurse using surgical asepsis? Applying a gown and gloves Maintaining objects and areas free from pathogenic microorganisms Handling hazardous and infectious materials Reducing the number of organisms and preventing their spread

Maintaining objects and areas free from pathogenic microorganisms Rationale: Surgical asepsis involves maintaining objects and areas free from pathogenic microorganisms. Medical asepsis, or clean technique, involves reducing and preventing the spread of microorganisms, handling hazardous and infections materials, and applying a gown and gloves. Option one does not describe if the gown/and gloves are sterile.

A nurse caring for a client with leukemia who is undergoing chemotherapy reviews the latest laboratory results and notes that the neutrophil count is below 500 cells/mm3 (0.5 x 109/L). Which of the following interventions does the nurse implement on the basis of this finding? Select all that apply. Monitoring the client's oral temperature Avoiding overinflation of the blood pressure cuff and rotating the cuff among several sites when measuring the blood pressure Requiring the client to use an electric shaver rather than a razor Maintaining sterile occlusion of intravenous (IV) catheters Providing a soft toothbrush for oral care Performing meticulous skin decontamination before venipuncture

Monitoring the client's oral temperature Maintaining sterile occlusion of intravenous (IV) catheters Performing meticulous skin decontamination before venipuncture Rationale: When the neutrophil count falls below 500 cells/mm3 (0.5 × 109/L), the client is at risk for infection. Monitoring of the oral temperature, maintaining sterile occlusion of IV and central venous catheters, and meticulous skin decontamination before venipuncture are critical nursing interventions for the client at risk for infection. The remaining options are interventions that are necessary for the client who has a low platelet count and is at risk for bleeding.

A home care nurse is visiting an older client who has been recovering from a mild brain attack (stroke) affecting her left side. The client lives alone but receives regular assistance from her daughter and son, who both live within 10 miles. Which of the following actions should the nurse take to assess the client's safety risk? Select all that apply. Observe the client's gait and posture Request that the client transfer to an assisted living environment for at least 1 month Ask a family member to move in with the client until her recovery is complete Evaluate the client's muscle strength Look for any hazards in the home environment Assess the client's visual acuity

Observe the client's gait and posture Evaluate the client's muscle strength Look for any hazards in the home environment Assess the client's visual acuity Rationale: To conduct a thorough client assessment, the nurse looks for risk factors related to safety. The assessment should include the assessment of visual acuity, gait and posture, and muscle strength, because alterations in these areas increase the client's risk for falls and injury. The nurse should also assess the home environment, looking for any hazards or obstacles that might affect safety. Asking a family member to move in with the client until recovery is complete and requesting that the client transfer to an assisted living environment for at least 1 month are not assessment activities. Additionally, nothing in the question indicates that these actions are necessary; therefore, these options are unrealistic and unreasonable.

A nurse is preparing a continuous intravenous (IV) infusion at the medication cart. As the nurse goes to attach the IV tubing port to the solution bag, the tubing drops, hitting the top of the medication cart. Which action should the nurse take to maintain asepsis? Scrub the tubing port with an alcohol swab Obtain a new IV solution bag Obtain new IV tubing Wipe the tubing port with povidone-iodine solution

Obtain new IV tubing Rationale: If IV tubing becomes contaminated as a result of coming into contact with some nonsterile object, the nurse should obtain new IV tubing. Contaminated tubing could cause systemic infection in the client. The IV solution bag has not been contaminated and does not need replacement. Wiping the tubing port with Betadine or scrubbing it with alcohol is insufficient and would be contraindicated regardless, because the tubing will be attached directly to a catheter in the client's vein.

A nurse leading an educational session about terrorism for members of the community is discussing anthrax. Which of the following pieces of information should the nurse provide to the group attending the session? Select all that apply. Anthrax can be transmitted from person to person. One way that anthrax can be contracted is through the skin. Anthrax is never fatal. A blood test is available for the detection of anthrax. No vaccine to prevent anthrax is available.

One way that anthrax can be contracted is through the skin. A blood test is available for the detection of anthrax. Rationale: Anthrax, which is caused by Bacillus anthracis, can be contracted through the digestive system or abrasions in the skin or by way of inhalation. In the lungs, anthrax can cause a buildup of fluid, tissue decay, and death; untreated pulmonary anthrax is fatal. A blood test performed to detect anthrax magnifies DNA from the blood sample and matches it to anthrax DNA. A vaccine exists, but its availability is limited. Anthrax is usually treated with ciprofloxacin, doxycycline, or penicillin.

A nurse caring for a client who is under airborne precautions notes that the client is scheduled for a nuclear scan. Which action on the part of the nurse is appropriate? Planning to have the nuclear scan performed at the bedside Asking the technicians in the nuclear scan department to wear masks Calling the nuclear medicine department and telling the technician that the test will have to be delayed until airborne precautions have been discontinued Placing a surgical mask on the client for transport and for contact with other individuals

Placing a surgical mask on the client for transport and for contact with other individuals Rationale: If the client is under airborne precautions, client movement and transport should be limited as much as possible. If transport or movement is necessary, the nurse can minimize the dispersal of droplet nuclei from the client by placing a surgical mask on the client. Having the scan performed at the bedside is unreasonable. Asking the technicians in the nuclear medicine department to wear masks would not prevent the dispersal of droplet nuclei from the client. The primary health care provider is the individual who would prescribe the cancellation or delay of a diagnostic test. Additionally, delaying the test until airborne precautions have been discontinued is not within the role of the nurse.

A nurse, preparing a sterile field on which to perform a dressing change, places the sterile drape on the overbed table. Which of these actions on the part of the nurse indicate correct understanding of the principles of aseptic technique? Select all that apply. Positioning the sterile field so that it remains in full view Holding the pair of sterile forceps below waist level area Picking up a pair of sterile scissors from the sterile field with a sterile gloved hand Pouring sterile wound cleansing solution into a sterile cup before donning sterile gloves Leaving the room to obtain a bottle of sterile normal saline solution Reaching across the sterile field to pick up a sterile gauze

Positioning the sterile field so that it remains in full view Picking up a pair of sterile scissors from the sterile field with a sterile gloved hand Pouring sterile wound cleansing solution into a sterile cup before donning sterile gloves Rationale: View Video. The principles of surgical asepsis must be followed in the preparation of a sterile field. Among these principles: A sterile object remains sterile only when touched by other sterile objects; only sterile objects may be placed on a sterile field; a sterile object or field out of the range of vision or an object held below the nurse's waist is to be considered contaminated; a sterile object or field becomes contaminated with prolonged exposure to air; when a sterile surface comes in contact with a wet, contaminated surface, the sterile object or field becomes contaminated by way of capillary action; fluid flows in the direction of gravity; a 1-inch edge of a sterile field or container is to be considered contaminated.

A nurse caring for a 9-month-old who has undergone repair of a cleft palate applies elbow restraints to the child. The mother visits her child and asks the nurse to remove the restraints. According to the guidelines for the use of restraints, what should the nurse do in response to the mother's request? Remove a restraint from one extremity Tell the mother that the restraints may not be removed Remove both restraints Loosen the restraints after telling the mother that they may not be removed

Remove a restraint from one extremity Rationale: Elbow restraints are used after cleft palate repair to prevent the child from touching the repair site, which could cause rupture or tearing of the sutures. The restraints may be removed one at a time only with a parent or nurse in constant attendance. Removing both restraints, telling the mother that the restraints may not be removed, and loosening the restraints are all incorrect nursing actions.

The safety department is providing a yearly educational session on fire safety and the use of fire extinguishers. A nurse is asked to demonstrate the use of a fire extinguisher after the session. What does the nurse do first when demonstrating appropriate use of the fire extinguisher? Squeezing the handle of the extinguisher Sweeping from the top to the bottom of the fire with the extinguisher Pulling the pin on the fire extinguisher Aiming at the base of the fire

Pulling the pin on the fire extinguisher Rationale: To use a fire extinguisher, pull the pin first. Next, aim the extinguisher at the base of the fire. Squeeze the handle of the extinguisher, then extinguish the fire by sweeping from side to side to coat the area evenly.

A home health nurse is performing an assessment of a client's skin. The nurse, noting multiple threadlike lines, both straight and wavy, beneath the skin, recognizes the presence of scabies. Which of the following precautions should the nurse institute before completing the assessment of the client? Avoiding sitting on the client's furniture Putting on a gown and gloves Putting on a pair of gloves Donning a mask and gloves

Putting on a gown and gloves Rationale: Scabies is usually transmitted from person to person by way of direct skin contact. The Centers for Disease Control and Prevention recommends the wearing of gowns and gloves for close contact with a person infested with scabies. Masks are not necessary. Transmission by way of clothing and other inanimate objects is uncommon. Everyone with whom the client has had contact should be treated for scabies at the same time.

A nurse employed in a physician's office hears a client in the waiting room call out, "Help! Fire!" The nurse rushes to the waiting room and finds that the wastebasket is on fire. What should the nurse immediately do? Remove the clients from the waiting room Confine the fire Extinguish the fire Activate the fire alarm

Remove the clients from the waiting room Rationale: The nurse should immediately remove any clients in immediate danger. The next step is activating the fire alarm. The nurse would then confine the fire by closing all of the doors and, finally, extinguish the fire. Note the strategic word "immediately." This indicates the first priority the nurse must take.

A nurse is admitting a postoperative client from the post-anesthesia care unit (PACU)to the surgical nursing unit. Which of the following measures should the nurse take for the safety of the client? Putting the side rails up after moving the client from the stretcher Quickly moving the client from the stretcher to the bed Uncovering the client before making the transfer from the stretcher to the bed Asking the client to slide from the stretcher to the bed

Putting the side rails up after moving the client from the stretcher Rationale: Because the client may be experiencing residual effects of anesthesia, the nurse should raise the side rails after transferring the client from the stretcher to the bed. Agency policy for the use of side rails is always followed; some agencies' policies set forth the number of side rails that may be used. Because of the effects of anesthesia and postoperative pain, it is not realistic to ask the client to slide from the stretcher to the bed. Hurried movements and rapid changes in position should be avoided because they may trigger orthostatic hypotension. The nurse should avoid exposing the postoperative client during transfer because of the potential for heat loss, respiratory infection, and shock.

The nurse administers a dose of ramipril 2.5 mg to a client at 9 am. While documenting administration of the medication, the nurse discovers that 1.25 mg, not 2.5 mg, was the prescribed dose. The nurse assesses the client, completes an incident report, and notifies the primary health care provider and nursing supervisor of the error. What statement does the nurse add to the client's record? Client's blood pressure was 128/82 mm Hg after the administration of the incorrect dose of ramipril. An incident report was completed and filed. Twice the amount of the prescribed ramipril was administered at 9 am. Ramipril 2.5 mg was administered at 9 am.

Ramipril 2.5 mg was administered at 9 am. Rationale: After an incident, the nurse would document a concise and objective description of what occurred and any follow-up actions taken in the client's record. The nurse would not document in the client's record that an incident report was completed. Nor would the nurse document that twice the prescribed dose was given or that an incorrect dose was given.

A client with paraplegia has spasticity of the leg muscles. Which nursing interventions should be included in the plan of care for this client? Select all that apply. Range-of-motion exercises of the affected limbs Removal of potentially harmful objects near the client An as-needed prescription for a muscle relaxant The use of padding against the client's legs when the client is sitting in a wheelchair The use of restraints to immobilize the limbs

Range-of-motion exercises of the affected limbs Removal of potentially harmful objects near the client An as-needed prescription for a muscle relaxant The use of padding against the client's legs when the client is sitting in a wheelchair Rationale: The use of limb restraints will not alleviate spasticity and could harm the client, so restraints should be avoided. Range-of-motion exercises are beneficial in stretching the muscles, which may diminish spasticity. The use of muscle relaxants may be helpful if spasms are causing discomfort for the client or pose a risk to the client's safety. Removing potentially harmful objects from the vicinity of the client is a good basic safety measure. Padding will prevent injury to the client's legs while the client is in the wheelchair.

Which actions should the nurse take in the event of an accidental poisoning? Select all that apply. Removing any visible materials from the nose and mouth Determining the type and amount of substance ingested Inducing vomiting if a household cleaner has been ingested Saving vomitus for laboratory analysis Assessing the client's airway patency, breathing, and circulation Placing the client in the supine position

Removing any visible materials from the nose and mouth Determining the type and amount of substance ingested Saving vomitus for laboratory analysis Assessing the client's airway patency, breathing, and circulation Rationale: In the event of accidental poisoning, the poison center is called before any attempt at interventions is made. Additional interventions in an accidental poisoning include assessing the client's airway patency, breathing, and circulation; removing any visible materials from the nose and mouth to terminate exposure; determining the type and amount of substance ingested, if possible, to identify an antidote; saving vomitus for laboratory analysis, which may aid further treatment; and positioning the victim with the head to the side to prevent aspiration of vomitus and help keep the airway open. Because of the risk of aspiration, vomiting is never induced in an unconscious client or in a client who is experiencing seizures. Additionally, vomiting is not induced if lye, a household cleaner, a hair care product, grease, a petroleum product, or furniture polish has been ingested, because of the risk of internal burns.

A community health nurse is providing information to local residents about the transmission of anthrax. Through which body systems does the nurse tell the residents that anthrax can be contracted? Select all that apply. Lymphatic Respiratory (Lungs) Integumentary System (Skin) Gastrointestinal Urinary Immune

Respiratory (Lungs) Integumentary System (Skin) Gastrointestinal Rationale: Anthrax, caused by Bacillus anthracis, can be contracted through the respiratory system (inhalation), the gastrointestinal system, and the integumentary system (skin). . It is not contracted through the immune system, urinary tract, or lymphatic system.

A staff nurse caring for a client with a head injury notes that the client is restless and pulling at the intravenous (IV) line. The client's primary health care provider does not want to prescribe sedation, and the family has requested that the client not be restrained. Which action by the nurse is best? Asking a family member to sit with the client Telling the family that the application of wrist restraints is critical in preventing injury to the client Asking a nursing assistant to monitor the client Staying with the client and consulting with the nurse manager about the situation

Staying with the client and consulting with the nurse manager about the situation Rationale: The nurse must stay with the client and consult with the nurse manager about the situation. It may be necessary for the nurse manager to call the supervisor to request an additional staff member to care for the client. Because the client has a head injury, the development of increased intracranial pressure (ICP) is a major concern. A nursing assistant is not trained to monitor the client for increased ICP. It is inappropriate to ask a family member to sit with the client. The application of restraints may agitate the client, causing further restlessness and thus increasing ICP.

A client with an infection is receiving antibiotics by way of intramuscular (IM) injection. The client is also receiving subcutaneous (SC) injections of heparin. Which precautions does the nurse understand are most appropriate to help ensure the safety of this client? Select all that apply. Switching injection sites Applying a pressure bandage to the site after each IM injection Decreasing the sizes of the needles used for the IM and SC injections Applying prolonged pressure to the sites of the IM and SC injections Doubling the dose of anticoagulant

Switching injection sites Applying prolonged pressure to the sites of the IM and SC injections Rationale: The use of anticoagulants puts the client at risk for bleeding. Prolonged pressure over the site of an IM injection will help prevent bleeding into the tissues surrounding the injection site. Switching injection sites can help decrease the risk of bruising and bleeding. Doubling the dose of anticoagulants is incorrect. Decreasing the needle sizes may be helpful but is not necessary. A pressure bandage is not an appropriate measure and is also unnecessary.

A nurse, charting the administration of medications to an assigned client at 9 pm, notes that atenolol was prescribed to be administered at 9 am instead of 9 pm. The nurse checks the client's vital signs, completes an incident report, and calls the primary health care provider to report the error. The primary health care provider tells the nurse that an incident report is not needed but instructs her to monitor the client during the night for hypotension. What action should the nurse take? Tell the primary health care provider that the error warrants the completion of an incident report Tell the nursing supervisor that the primary health care provider did not want an incident report completed and filed Tear up and discard the incident report Notify the nursing supervisor

Tell the primary health care provider that the error warrants the completion of an incident report Rationale: Incident reports are an important part of a healthcare agency's quality improvement program. An incident is any event that is not consistent with the routine operation of a healthcare unit or routine care of a client. An example of an incident is administering a medication at a time at which it is not prescribed to be given. Whenever an incident occurs, an incident report is completed and filed in accordance with agency guidelines. The nursing supervisor would be notified of the incident; however, on the basis of the data in the question, the nurse should tell the primary health care provider that the error warrants completion and follow-through with an incident report. Therefore, the other options are incorrect.

A nurse is assisting with disaster relief after a tornado. The nurse's goal with the overall community is to prevent as much injury and death resulting from the uncontrollable event as possible. Finding safe housing for survivors, providing support to families, organizing counseling sessions, and securing physical care when needed are all examples of which level of prevention? Secondary Primary Initial Tertiary

Tertiary Rationale: Tertiary prevention involves the reduction of the amount and degree of disability, injury, and damage after a crisis. Primary prevention is aimed at keeping a crisis from ever occurring, and secondary prevention is focused on reducing the intensity and duration of the crisis during the actual crisis. There is no such thing as the initial prevention level.

A nurse provides instruction to a new nursing assistant regarding the application of a restraint to a client. The nurse watches as the nursing assistant applies the restraint. What observation tells the nurse that the nursing assistant is using correct procedure? The assistant secures the restraint in such a way that it is impossible to slip a finger between the restraint and the client's skin. The assistant applies a tie knot in the restraint strap. The assistant attaches the restraint straps securely to the side rails. The assistant applies the restraint so that the strap does not tighten when force is applied against it.

The assistant applies the restraint so that the strap does not tighten when force is applied against it. Rationale: A half-bow or safety knot should be used to apply a restraint, because it does not tighten when force is applied against it and because it allows quick, easy removal of the restraint in the event of an emergency. The restraint strap is secured to the bed frame, never to the side rails, to help prevent accidental injury in the event that the side rail is released. A restraint should be secured in such a way that one or two fingers can be easily slipped between the restraint and the client's skin.

A registered nurse is instructing a group of nursing assistants in the principles of body mechanics. Which of these observations tell the nurse that a student is using the principles appropriately? Select all that apply. The assistant positions a box that is to be lifted between his knees. The assistant helps a client requiring total care into a chair without additional assistance. The assistant turns his back to change position while moving a client. The assistant leans forward when turning a client in bed. The assistant keeps the object to be moved as close to his body as possible.

The assistant positions a box that is to be lifted between his knees. The assistant keeps the object to be moved as close to his body as possible. Rationale: When moving an object, the nursing assistant should position the object between his knees. The assistant should keep the client or object to be moved as close to his body as possible. When turning a client, the assistant should keep his back straight and take small steps with the feet. The assistant should turn his feet, rather than twisting his back, if a change in direction is necessary when carrying an object or a client. The assistant should seek out assistance when transferring a client who requires total care.

After discussing the use of restraints with a client and family, the primary health care provider has written a prescription for wrist restraints to be applied to a client. The nurse instructs the nursing assistant to apply the restraints. Which of the following observations by the nurse indicates that the nursing assistant is using the restraints safely and correctly? Select all that apply. The restraints have been tied to the side rails of the bed. The call light has been placed within reach of the client. A safety knot has been used to secure the restraints. The restraints are applied tightly. The restraints are being released every 2 hours.

The call light has been placed within reach of the client. A safety knot has been used to secure the restraints. The restraints are being released every 2 hours. Rationale: Restraints should never be applied tightly, because this could impair circulation. They should be tied to the bed frame (not the side rail) with the use of a safety knot. The client could sustain injury if the side rail were lowered with a restraint attached to it. A safety knot is used because it can easily be released in an emergency. Restraints must be released every 2 hours to facilitate inspection of the skin, help ensure good circulation, and permit movement of the joint through its range of motion. The call light must always be within reach of the client in case he or she needs assistance.

A home health nurse has been called to the home of an older postoperative cardiovascular client by the client's son. The son tells the nurse, "We're using a hospital bed here at home, but my mother has fallen out of bed three times." Which observation by the nurse reflects an increased risk of this client's falling out of bed? The caregiver leaves both side rails down while the client is in bed. The client is oriented to person, place, and time. The client's bed is in a low position. The caregiver uses the overbed table for feedings.

The caregiver leaves both side rails down while the client is in bed. Rationale: Leaving the side rails of older client's bed down may increase the client's risk of falling. The aging process also increases this client's potential for falls; therefore, evaluating the safety of the environment is a necessity. Keeping the client's bed in a low position, orientating the client to the environment, and using the overbed table for feedings are all ways to help ensure the client's safety.

Wrist restraints have been prescribed for a client who is constantly pulling at his gastrostomy tube. Which findings does the nurse, developing a care plan, recognize as unexpected outcomes related to the use of restraints? Select all that apply. The client verbalizes the reason for the restraints. The client becomes agitated. The client slips his hand from its restraint and pulls at his gastrostomy tube. The client is unable to reach the gastrostomy tube with his hands. The client's left hand is pale and cold. The skin under the restraint is red.

The client becomes agitated. The client slips his hand from its restraint and pulls at his gastrostomy tube. The client's left hand is pale and cold. The skin under the restraint is red. Rationale: A physical restraint is a mechanical or physical device used to immobilize a client or extremity. The restraint restricts freedom of movement. Unexpected outcomes in the use of restraints include signs of impaired skin integrity, such as redness or skin breakdown; altered neurovascular status, such as cyanosis, pallor, coldness of the skin, or complaints of tingling, numbness, or pain; increased confusion, disorientation, or agitation; and escape from the restraint device that results in a fall or injury. Client verbalization of the reason for the restraints and the client's inability to reach the gastrostomy tube with his hands are expected outcomes.

A client with a new diagnosis of tuberculosis (TB) is being admitted to the hospital. During the collection of data from the client, which of the following considerations is most important? The name of the person from whom the client contracted TB, so that the person may be reported for follow-up care What medications have been prescribed and what the client knows about his or her side effects The religious affiliation or church of preference The names of close friends and family members

The names of close friends and family members Rationale: TB is a contagious disease that is spread in respiratory droplets. The nurse needs to elicit the names of close friends and family members so that these individuals may be tested for exposure to TB. The client's religious affiliation or church of preference is a component of the data collection process but is not the primary consideration of the options provided. It is premature to determine knowledge regarding medications, because treatment measures may not yet have been prescribed. Although knowing the name of the person from whom the disease was contracted would be helpful, the client may not know who it is; this is not the most important information the nurse needs to collect.

During a laboratory training session, the nurse is watching as a nursing assistant repositions a client. Which observation tells the nurse that further training is necessary? The nursing assistant encourages the client to assist as much as possible. The nursing assistant keeps his/her neck, back, pelvis, and feet aligned. The nursing assistant positions himself/herself close to the client. The nursing assistant keeps his/her knees straight and his/her feet close together.

The nursing assistant keeps his/her knees straight and his/her feet close together. Rationale: To help prevent injury, the nurse needs to use and encourage staff members to use good body mechanics and ergonomic principles in providing care. When planning to reposition a client, the staff member must assess the client's ability to assist and encourage the client to assist as much as possible. The nursing assistant should position himself or herself close to the client and keep the back, neck, pelvis, and feet aligned, avoiding twisting; use the arms and legs (not the back); and keep the knees flexed and the feet wide apart.

A nursing instructor is observing a nursing student who is practicing the use of standard precautions in the nursing laboratory. Which of the following observations by the instructor indicates a need for further teaching? The nursing student washes her hands before glove removal after emptying a Foley bag. The nursing student washes hands before making contact with the client. The nursing student changes gloves between tasks and procedures. The nursing student wears a gown to change the bed of an incontinent client.

The nursing student washes her hands before glove removal after emptying a Foley bag. Rationale: Standard precautions require that gloves be removed promptly after use and before the wearer touches noncontaminated surfaces or other clients. Gloves are not washed before removal, because splashing of contaminated material may result. Changing gloves between tasks and procedures, washing the hands before making contact with the client, and wearing a gown to change the bed of an incontinent client reflect correct understanding of the principles of standard precautions.

At the beginning of the 7 am-3 pm shift, the nurse checks her assigned clients and notes that a client with diabetes mellitus has an intravenous (IV) bag of 5% dextrose in water hanging and infusing instead of the prescribed 0.9% normal saline. The nurse verifies the prescription and changes the IV solution to the correct one. The nurse assesses the client noting that the blood glucose level at 7:15 am was 149 mg/dL (8.3 mmol/L), notifies the PCP, and completes an incident report. Which information about the event is appropriate for inclusion on the incident report? Select all that apply. INCIDENT REPORT - Events that Occurred The PCP was contacted. The blood glucose level at 7:15 am was 149 mg/dL. An IV solution of 5% dextrose in water was infusing at 7 am. A solution of 5% dextrose in water was infusing instead of the prescribed 0.9% normal saline solution. A 5% dextrose in water solution is not usually prescrib

The primary health care provider was contacted. The blood glucose level at 7:15 am was 149 mg/dL (8.3 mmol/L). An IV solution of 5% dextrose in water was infusing at 7 am. Rationale: The incident report should contain the client's name, age, and diagnosis. The report should contain a factual description of the incident, any injuries sustained by those involved, and the outcome of the situation. The nurse should avoid the use of subjective data, instead documenting objective data. The nurse also avoids any implication that an accident occurred or an error was made. The statement that a 5% dextrose in water solution was infusing instead of the prescribed 0.9% normal saline solution implies that an accident occurred or an error was made. Likewise, the statement that 5% dextrose in water is not usually prescribed for clients with diabetes and that the solution was changed immediately on its discovery makes an implication. The remaining statements identify factual and observable data without making implications.

A nurse educator is providing in-service sessions to the nursing staff regarding employee safety and the prevention of occupationally acquired HIV infection. Which of the following precautions does the nurse instruct the nursing staff to take as a means of preventing accidental needlesticks? Select all that apply. The use of needleless devices Disposal of needles in special puncture-resistant containers The use of latex gloves The use of shielded needles The use of recessed needles

The use of needleless devices Disposal of needles in special puncture-resistant containers The use of shielded needles The use of recessed needles Rationale: Although strict adherence to universal or standard precautions can reduce significantly the incidence of exposure to blood or body fluid, latex gloves cannot prevent a needlestick. The use of recessed needles, needleless devices, shielded needles, and puncture-resistant containers for the disposal of needles are all of significant benefit in the prevention of accidental needlesticks.

In which of the following situations would the nurse use this type of restraint (see figure)? Select all that apply. Image of patient in Mitt restraint To prevent the client from turning from side to side To prevent the client from getting out of bed To immobilize a client's arm and shoulders To secure the shoulders and the waist To prevent the use of the hands while allowing free arm movement To prevent dislodgment of an intravenous line

To prevent the use of the hands while allowing free arm movement To prevent dislodgment of an intravenous line Rationale: A mitten restraint is a thumbless device used to restrain the hands. It prevents the use of the hands while allowing free arm movement. Mitten restraints are useful for the client who must be prevented from dislodging an intravenous line, indwelling urinary catheter, nasogastric tube, other types of tubes, or wound dressings. A belt restraint prevents the client from falling out of a bed, a chair, or a stretcher. A mitten restraint does not secure the shoulders and the waist and is not used to prevent the client from turning side to side.

Contact precautions are initiated for a client with methicillin-resistant Staphylococcus aureus (MRSA) infection. What does the nurse, providing instructions to a nursing assistant about caring for the client, tell the assistant? That gloves only are needed to care for the client To wear gloves and a gown when changing the client's bed linen. To wear a gown when caring for the client and remove the gown immediately after leaving the client's room To transfer the client to a semiprivate room

To wear gloves and a gown when changing the client's bed linen. Rationale: Contact precautions require the use of gloves, gown, and goggles if direct client contact is anticipated. Goggles are worn to protect the mucous membranes of the eye during interventions that may produce splashes of blood or body fluids, secretions, or excretions. The client should be placed in a private room or, if a private room is not available, in a semiprivate room with another client who has active infection with the same microorganism but no other infection. The nursing assistant would remove the protective gear before leaving the client's room.

A nurse is reading the history and physical examination findings of an older client who has just been admitted to the hospital. Which findings documented in the history indicate an increased risk for accidents? Select all that apply. High-frequency hearing tones are perceptible. Transmission of hot impulses is delayed. The client's peripheral vision is decreased. Voluntary and autonomic reflexes are slowed. Increased lens accommodation The client complains of frequent nocturia.

Transmission of hot impulses is delayed. The client's peripheral vision is decreased. Voluntary and autonomic reflexes are slowed. The client complains of frequent nocturia. Rationale: The physiologic changes that occur during the aging process increase the client's risk for accidents. Musculoskeletal changes include diminished muscle strength and function, lessening of joint mobility, and limited range of motion. Nervous system changes include slowed voluntary and autonomic reflexes. Sensory changes include reduced peripheral vision and lens accommodation, delayed transmission of hot and cold impulses, and impaired hearing as high-frequency tones become less perceptible. Genitourinary changes include nocturia and incontinence.

A nurse is preparing to clean up a blood spill on the client's bedside table that occurred when a blood tube containing a specimen from the client broke. What steps should the nurse take to clean up the blood spill? Select all that apply. Using tongs to collect any broken glass Placing the pieces of broken glass in a plastic bag Disinfecting the area of the blood spill with a dilute bleach solution Blotting up the spill with a face cloth or cloth towel Wearing gloves for the cleanup procedure

Using tongs to collect any broken glass Disinfecting the area of the blood spill with a dilute bleach solution Wearing gloves for the cleanup procedure Rationale: The nurse should blot the spill with an absorbent disposable material such as disposable paper towels or terry wipes, not a face cloth or towel. Tongs are used to pick up any broken glass, and gloves are worn for the procedure. The broken glass is disposed of in a puncture-resistant container. The area is disinfected with a dilute bleach solution or other agency-accepted product.

A client undergoing chemotherapy is found to have an extremely low white blood cell count, and neutropenic precautions, including a low-bacteria diet, are immediately instituted. Which of these food items will the client be allowed to consume? Select all that apply. Well-cooked cheeseburger Tossed salad Italian bread Raw celery Fresh apple Baked chicken

Well-cooked cheeseburger Italian bread Baked chicken Rationale: An extremely low white blood cell count puts the client at risk for infection, necessitating the implementation of a low-bacteria diet. The client must avoid fresh fruits and vegetables, which may harbor microorganisms that could cause infection, and ensure that meat is thoroughly cooked. Italian bread, baked chicken, and a well-done cheeseburger are all acceptable foods for the client.

A nurse employed on a medical care unit is administering medications. She tells a client that she is going to administer his furosemide through his intravenous (IV) line. The client tells the nurse that he takes this medication orally at home every day and is concerned that it is being administered by way of a different route. The nurse should take which most appropriate action? Explaining to the client that the oral route will not permit the medication to exert an adequate effect Verifying the primary health care provider's prescription Sitting and talking to the client to alleviate his concern Letting the client know that most medications are administered by way of the IV route when a client is hospitalized

Verifying the primary health care provider's prescription Rationale: If the client questions a primary health care provider's prescription, the nurse must verify the prescription. This is the most appropriate action. Although it is appropriate to talk to the client and alleviate concerns, this is not the most appropriate action of those provided. Although in some client situations the IV route of administration of certain medications is more effective than the oral route, providing the client with this information is not the most appropriate action of the options provided. Critical care units in the hospital may administer most medications by way the IV route, but this is not necessarily the situation in a medical care unit.

A nurse giving a client a bed bath drops the towel on the floor. What should the nurse do? Borrow a towel from the client's roommate Use a bath blanket as a towel Wash his/her hands and go to the linen room to obtain another towel Wash his/her hands, pick up the towel, and shake the towel out

Wash his/her hands and go to the linen room to obtain another towel Rationale: To avoid spreading the client's germs, the nurse must wash her hands before leaving the client's room. Therefore the nurse should cover the client and ensure that the client is safe, wash her hands, and go to the linen room to obtain another towel. It is not appropriate to use a bath blanket as a towel. It is never appropriate to borrow other clients' supplies, because this is inconsistent with the principles of infection control. The nurse would never use linen that had been dropped on the floor. Also, shaking linen spreads germs.

Which of the following safety guidelines should the nurse include in the plan of care for a client with an internal radiation implant? Select all that apply. Wear a dosimeter film badge when in the client's room. Wear a lead shield when in the client's room. Keep all bed linens and dressings in the client's room until the implant is removed. Allow children to visit the client as long as they are at least 12 years old. Limit visits from family to 60 minutes per day.

Wear a dosimeter film badge when in the client's room. Wear a lead shield when in the client's room. Keep all bed linens and dressings in the client's room until the implant is removed. Rationale: Nursing responsibilities in the care of a client with an internal radiation implant, which involve preventing exposure to the radiation, include placing the client in a private room with a private bath; rotating nursing assignments and organizing nursing tasks to minimize exposure to the radiation source; limiting time to 30 minutes per care provider per shift; wearing a dosimeter film badge to measure radiation exposure; wearing a lead shield to reduce the transmission of radiation; not allowing pregnant women or children younger than 16 years to visit the client; limiting visitors to 30 minutes per day (visitors should stay at least 6 feet (2 meters) from the source); keeping all bed linens and dressings in the client's room until the implant is removed; keeping a lead container in the client's room for housing the implant if it should be dislodged; and avoiding touching a dislodged radiation source (long-handled forceps are used to place the source in the lead container).

Which of the following actions are in keeping with the principles of standard precautions? Select all that apply. Cleaning of blood spills with soap and warm water Handwashing before removal of a pair of soiled gloves Wearing a gown and gloves when changing the linens on the bed of a client with a draining lesion of the leg Discarding needles in puncture-resistant containers Handwashing between client contacts Wearing a face shield as a part of the protective garb during a wound irrigation

Wearing a gown and gloves when changing the linens on the bed of a client with a draining lesion of the leg Discarding needles in puncture-resistant containers Handwashing between client contacts Wearing a face shield as a part of the protective garb during a wound irrigation Rationale: Standard precautions must be practiced with all clients in every setting. These precautions involve handwashing and the use of gloves, masks, eye protection, and gowns, as well as other protective devices, when they are appropriate for client contact. These precautions apply to contact with blood, body fluids, nonintact skin, and mucous membranes. The hands are always washed between client contacts and after (not before) gloves are removed. Needles are not recapped (unless the agency provides special and agency-approved recapping devices for primary healthcare providers) and are discarded in puncture-resistant containers. Spills of blood or body fluids are cleaned up with a solution of bleach and water (diluted 1:10) or other agency-approved disinfectant. A mask, eye protection, or face shield is worn if client care activities have the potential to result in splashes or spraying of blood or body fluid. A gown is worn if soiling of clothing is likely.

A nurse receives a telephone call from the admissions office and is told that a client scheduled for an internal radiation implant will be admitted to the nursing unit. Which of the following precautions does the nurse include in the client's plan of care? Wearing gloves when emptying the client's bedpan Placing used linen in double bags and sending a bag to the laundry room every evening Allowing the client to ambulate in the hall only once a day Placing the client in a semiprivate room at the end of a hallway

Wearing gloves when emptying the client's bedpan Rationale: A primary goal of care for the client with an internal radiation implant is to prevent exposure of others to radiation. Therefore, a client with an internal radiation implant is required to remain in a private room to prevent accidental exposure of other clients, staff, and visitors to radiation. For this reason, a private room with a private bath is essential. All client linens should be kept in the client's room until the implant is removed. Wearing gloves when emptying the client's bedpan is the only appropriate intervention, of those provided, for a client with an internal radiation implant.

A nurse is preparing a chemotherapy infusion to be administered to a client with a diagnosis of Hodgkin's disease. Which precaution should the nurse take while working with this intravenous (IV) infusion? Wearing gloves and a mask Wearing gloves and a gown Wearing gloves, a mask, and a head covering Wearing gloves, a mask, and eye protection

Wearing gloves, a mask, and eye protection Rationale: When handling chemotherapeutic agents, the nurse should wear disposable latex gloves, a mask that covers the nose and mouth, and eye protection, especially if a biological hood is not available. Wearing gloves and a mask or gloves and a gown will not provide adequate protection. A head covering is not necessary.

Which event would require a nurse to complete and file an incident report? The nurse, preparing an intravenous infusion, notes that the battery of an intravenous infusion pump is not working. A client has a seizure. When a visitor suddenly becomes weak and dizzy, the nurse checks the visitor's blood pressure and takes the visitor to the emergency department for treatment. The nurse determines that a client would benefit from the use of a walker to ambulate.

When a visitor suddenly becomes weak and dizzy, the nurse checks the visitor's blood pressure and takes the visitor to the emergency department for treatment. Rationale: An incident is any event that is not consistent with the routine operation of a healthcare unit or routine care of a client. Examples of incidents include client falls, needlestick injuries, a visitor having signs/symptoms of illness, medication administration errors, accidental omission of prescribed therapies, and circumstances leading to injury or a risk for injury. An incident report does not need to be filed if a client has a seizure unless the client sustains injury as a result of the seizure. If the nurse determines that a client would benefit from the use of a walker to ambulate, he or she should take the appropriate action to obtain one. If the nurse notes that the battery of an intravenous infusion pump is not working, he/she should obtain a functioning pump and send the nonfunctioning pump to the appropriate department for repair.

A nurse performs an evaluation to determine whether a client's home is electrically safe. Which finding indicates the need for further investigation and intervention? A safety-type extension cord is secured to the floor with electrical tape. Electrical kitchen appliances are located away from the sink. Wiring for the television runs under the carpet. Electrical cords are free of frayed and damaged wires.

Wiring for the television runs under the carpet. Rationale: Electrical safety guidelines must be followed to help prevent fires and injuries. These guidelines include the maintaining of electrical equipment in good working order with proper grounding; periodically checking electrical cords and outlets for exposed, frayed, or damage wires and loose or missing parts; avoiding overload of electrical circuits; reading warning labels on all equipment; never operating unfamiliar equipment; using safety-style extension cords and using such cords only when absolutely necessary, securing them to the floor with electrical tape; never running electrical wiring under a carpet; never pulling a plug by the cord; never using electrical appliances near sinks, bathtubs, or other water sources; and disconnecting a plug from the outlet before cleaning the equipment or appliance to which it is attached.

A home care nurse is instructing a client in the use of ice packs to treat an eye injury. What does the nurse instruct the client to do? Wrap a plastic bag filled with ice in a pillowcase and place it on the eye Avoid the use of commercially prepared ice bags Place the ice pack directly on the eye Keep the ice pack on the eye continuously for 24 hours

Wrap a plastic bag filled with ice in a pillowcase and place it on the eye Rationale: An ice pack placed directly against the skin or left in place for an extended period carries a risk of tissue damage similar to that of a hot water bottle. To help prevent tissue damage resulting from excessive cold exposure, the ice pack should be removed in most cases after 30 minutes; after a short time it may be reapplied. An ice pack should never be placed directly against the skin; instead, it should be covered with a pillowcase or towel. Commercially prepared ice bags are appropriate for use as ice packs.


Kaugnay na mga set ng pag-aaral

Communications 103 Final Study Guide (Richardson)

View Set

Chapter 1-8 Business 1301 midterm review

View Set

A Prayerful Response: From Adoration to Surrender

View Set

Health Assessment Chapters 16 & 17

View Set

Money Management Final Review: MC

View Set